JEE Main & Advanced Mathematics Complex Numbers and Quadratic Equations Question Bank Conjugate, Modulus and Argument of complex number

  • question_answer
    Argument of \[-1-i\sqrt{3}\] is [RPET 2003]

    A) \[\frac{2\pi }{3}\]

    B) \[\frac{\pi }{3}\]

    C) \[-\frac{\pi }{3}\]

    D) \[-\frac{2\pi }{3}\]

    Correct Answer: D

    Solution :

    Let \[z=-1-i\sqrt{3}\] then\[\alpha ={{\tan }^{-1}}\left| \,\frac{b}{a}\, \right|={{\tan }^{-1}}\left| \,-\frac{\sqrt{3}}{1}\, \right|=\frac{\pi }{3}\] Clearly, z is in III quadrant. Therefore argument \[\theta =-(\pi -\alpha )=-(\pi -\pi /3)=\frac{-2\pi }{3}\].


You need to login to perform this action.
You will be redirected in 3 sec spinner